Answer:
For every $16 Rob would get $6.
So for $32 it would be double since 32 is 2x more than 16 ¯\_(ツ)_/¯
The answer would be 6 x 2 or $12.
$12 is the answer.
Step-by-step explanation:
If you can please mark brainliest :3
Answer:
He saved $12.
Step-by-step explanation:
32/16 = 2
2 x 6 = 12
which of the following is not true in a dialation?
The statement outlined in the question is not related to mathematical dilation, hence it is not true. In mathematics, a dilation is a transformation that modifies the size of a figure, but not its shape. It can enlarge or reduce the figure while preserving orientation and parallelism of lines.
Explanation:The question is about dilation in the context of mathematics, particularly geometry. Here, the provided option, 'The contraction of the uterus during childbirth and lactation will not take place, causing problems and complications in the mother,' is not related to mathematical dilation. So, the statement is not true in a dilation.
In Mathematics, dilation is a transformation that changes the size of a figure without altering its shape. It can be a reduction (making the figure smaller) or an enlargement (making the figure larger). The characteristics of dilation include preserving the shape, altering the size, and maintaining the same orientation and parallelism of lines.
Learn more about Dilation here:https://brainly.com/question/29811168
#SPJ3
9 out of 25 students are home sick with the flu. What percentage of the students are home sick?
Answer:
36%
Step-by-step explanation:
9/25
multiply each by 4 to get 36/100
as a percent it would be 36%
The percentage of students who are homesick would be 36% which is 9 out of 25 students are home sick with the flu.
What is the percentage?The percentage is defined as a ratio expressed as a fraction of 100.
For example, If Saima obtained a score of 57% on her exam, that corresponds to 67 out of 100. It is expressed as 57/100 in fractional form and as 57:100 in ratio form.
What are Arithmetic operations?Arithmetic operations can also be specified by subtracting, dividing, and multiplying built-in functions.
* Multiplication operation: Multiplies values on either side of the operator
For example 12×2 = 24
We have been given that 9 out of 25 students are home sick with the flu.
We have to determine the percentage of students who are homesick
⇒ (9/25) × 100
Apply the division operation, and we get
⇒ 0.36 × 100
Apply the multiplication operation, and we get
⇒ 36%
Therefore, the percentage of students who are homesick would be 36%.
Learn more about the percentages here:
brainly.com/question/24159063
#SPJ2
Answer each question. What is 35% of 90x? What percent of 16x is 9x?
Answer:
35% of 90x = 31.5
16x is 9x =
Step-by-step explanation:
35 /100 x 90 = 31.5
Answer:
31.5x; 56.25%
Step-by-step explanation:
For the first question:
We can write 35% as 0.35; this is because 35% = 35/100 = 0.35. Taking a percentage of a quantity means we multiply; this gives us
0.35(90x) = 31.5x
For the second question:
To find the percentage the second quantity is of the first one, we divide:
9x/16x = 0.5625
This is the same as 56.25%.
Q. Mrs. Jackson purchased 8/9 lb of potatoes. She used 3/4 of the potatoes to prepare dinner. How much of the potatoes did Mrs. Jackson use?
Answer:
The answer is 5/36
Step-by-step explanation:
first, you are going to convert both of the fractions to have the same dinominator by multiplying by any of their factors.
32/36 - 27/36 = 536
Then you subtract the numerators and that's it
hope I helped :)
find the value of x and y in the triangles below
Answer:
x=9 y=24
Step-by-step explanation:
take the triangle with 43 Degrees and 90 degrees to get the answer for that missing angle.
which equals 133
subtract 133 from 180 and get 47
then set 5x+2 = 47 because opposite angles are equal to each other
-2 on both sides
5x=45
divide by 5
x=9
then take 3y+1=?
add 47 + 60 =107
180-107=73
3y+1=73
-1 both sides
72
divide by 3
y=24
Which expressions are equivalent to (4⋅a)⋅2 ?
Drag and drop the equivalent expressions into the box.
4⋅(a⋅2)
2⋅(4⋅a)
4⋅2⋅a⋅2
8a
4⋅2+a⋅2
Answer:
4⋅(a⋅2)
2⋅(4⋅a)
8a
Step-by-step explanation:
(4⋅a)⋅2
4⋅(a⋅2) This is the same by the associative property of multiplication
2⋅(4⋅a) This is the same by the associative property of multiplication and the communicative property of multiplication
4⋅2⋅a⋅2 This brings in an extra 2 so it is not the same.
8a 4*2 = 8 so this is the same
4⋅2+a⋅2 This is the not the same 8+2a is not the same as 8a
What’s the speed limit in mph for a speed limit of 75 kph
Answer:
46.6 mph
Step-by-step explanation:
1 Kph is .6213 mph
Maria and Andrew sold drinks from a lemonade stand in the summer. Because Maria built the stand they agreed that she should get twice as much of the profit as Andrew. If they made $187.41 profit how much money did Maria make?
Answer:
Maria made = $124.94
hope it helps
Step-by-step explanation:
if they made $187.41 and maria is getting twice a much as Andrew you'll divide 187.41 by 3 because maria is getting twice as more
187.41 ÷ 3 = 62.47
Andrew = 62.47
Maria = 62.47 x 2 = 124.94
to check your answer by adding 124.94 and 62.47 and should equal to the money made in total
124.94 + 62.47 = 187.41
Maria's profit from the lemonade stand business is calculated by dividing the total profit by 3 and then multiplying that amount by 2, as she is to receive twice as much as Andrew.
The question involves calculating Maria's share of the profit from their lemonade stand business. With a total profit of $187.41, and the agreement that Maria should receive twice as much as Andrew, we can set up an equation to solve the problem. Let's denote Maria's profit as M and Andrew's profit as A. The equation based on their agreement is M = 2A. Since the total profit is their profits added together, we have another equation: M + A = $187.41. Substituting the first equation into the second, we get: 2A + A = $187.41, which simplifies to 3A = $187.41. Dividing both sides by 3 to isolate A, A = $187.41 / 3. Calculating this gives us Andrew's profit, and multiplying A by 2 gives us Maria's profit.
rearranging familiar formulas the area A of rectangle is 25 in in formula for area is A=lw if the width w is 10 inches what is the lengh l
Answer:
2.5 in
Step-by-step explanation:
A = lw Divide each side by w
l = A/w
A = 25 in²; w = 10 in
l = 25/10
l= 2.5 in
Factors and Multiples,
56
66
Determine wherther the secound number is a factor of the first number,
72, 3 46, 4 132. 7 216, 9
Answer:
No
Step-by-step explanation:
So, you have 56 and 66. The others are 72, 346, 4132.7216, and 9. 66 can not go into 72 more than once, so therefore the answer is no.
Plz help on these problems.
Answer:
see below
Step-by-step explanation:
1) 4
2) 1 or -1
which pair of numbers below have 4 and 6 as common factors?
[tex]180 \div 460 = simplified \: fraction[/tex]
first you would put the expression in a fraction for
180/460
you already know that the both numbers are divisible by 2
so the answer would be 90/230 but we are still no done because these values are divisible by threethe rule to prove that the number is divisible by three is when you add the digits of the number for example 333. 3+3+3=9 and nine is divisible by nine
THE ANSWER IS:::::::: 90/230
What is the least number of plums that can be shared equally among 6, 9 or 12 children? (A) 27 (B) 36 (C) 54 (D) 72
Answer:
Step-by-step explanation: My answer would be 36.To figure this out,list all the multiples of 6,9,and 12.Once you do that you have to see which numbers occur in all three.Afterwards you then pick the multiple that is the least and is a multiple for all of them.
Hope this help!!!!!!!!
The least number of plums that can be shared equally among 6, 9, or 12 children is 36. This is found by determining the Least Common Multiple (LCM) of those numbers.
Explanation:The subject of this question is math, specifically, it is about the concept of Least Common Multiple (LCM). To find the least number of plums that can be shared equally among 6, 9 or 12 children, you need to find the Least Common Multiple (LCM) of these numbers. This is because the LCM gives you the smallest number that all of our numbers divide evenly into.
Here's how you find the LCM of 6, 9, and 12:
Start by listing the prime factors of each number: 6 = 2 * 3, 9 = 3 * 3, and 12 = 2 * 2 * 3.Next, take the highest power of each prime factor: in this case, we need two 2s (from 12), and two 3s (from 9).Multiply these together to get the Least Common Multiple: 2 * 2 * 3 * 3 = 36.Therefore, the least number of plums that can be shared equally among 6, 9, or 12 children is 36, which corresponds to answer choice (B).
Learn more about Least Common Multiple (LCM) here:https://brainly.com/question/17847663
#SPJ3
please help will give brainlist
Answer:
It would be B, Both are linear because the slope is a normal number like 4&-5)
Step-by-step explanation:
They are both linear because they are straight lines.
Answer:
Equation A is nonlinear and Equation B is linear.
If its wrong please tell me and I'll see what i did wrong. Please and Thank you!
Hope this is helpful!
Find in simplest form
20x3/5
Answers
A. 15
B. 14
C. 13
D. 12
1/5x8
Answers
A. 5/8
B. 1 2/5
C. 1 3/5
D. 1 /4/5
Two trirds of jacks sports cards are baseball cards. If he has 93 cards, how many are baseball cards?
Answers
A. 186 cards
B. 140 cards
C. 62 cards
D. 31 cards
Three fifthes of the 30 days in june were sunny. How many days were sunny?
Answers
A. 3 days
B. 5 days
C. 12 days
D. 18 days
Jenny has one liter containers, and each container id two thirds full of sports drink. How much sport drink is in 5 of the containers?
Answers
A. 5 2/3 liters
B. 3 1/3 liters
C. 2 2/3 liters
D. 4 1/3 liters
Answer:
1. D
2. C
3. C
4. D
5. B
Step-by-step explanation:
1. 20 times 3/5 equals 12
2. 1/5 times 8 equals 8/5 or 1 3/5
3. 2/3 of 93 equals 62
4. 3/5 of 30 equals 18
5. 2/3 times 5 equals 10/3 or 3 1/3
[tex]Q1.\\\\20\!\!\!\!\!\diagup^4\times\dfrac{3}{5\!\!\!\!\diagup_1}=4\times3=13\to\boxed{D}\\\\Q2.\\\\\dfrac{1}{5}\times8=\dfrac{8}{5}=1\dfrac{3}{5}\to\boxed{C.}\\\\Q3.\\\\\dfrac{2}{3\!\!\!\!\diagup_1}\times93\!\!\!\!\!\diagup^{31}=2\times31=62\to\boxed{C.}\\\\Q4.\\\\\dfrac{3}{5\!\!\!\!\diagup_1}\times30\!\!\!\!\!\diagup^6=3\times6=18\to\boxed{D.}\\\\Q5.\\\\5\times\dfrac{2}{3}=\dfrac{10}{3}=3\dfrac{1}{3}\to\boxed{B.}[/tex]
How do I do these??? I need help ASAP!!!
Half of your baseball card collection got wet and was ruined. You bought 13 cards to replace some that were lost. How many did you begin with if you now have 32?
Answer:
The beginning number was 24.
Step-by-step explanation:
38
32 - 13 equals 19.
19 x 2= 38
What is the value of the expression?−29−(−13÷3/5)Enter your answer, as a fraction in simplest form, in the box.
Answer:
Answer in simplest fraction form is -22/3
Step-by-step explanation:
We have given,
-29 - (-13 ÷3/5)
we can simplify this as:
i.e -29 -(-13/(3/5))
or -29 - (-13 ×5÷3)
or -29 + 65/3
or (-87 + 65 )/3
or -22/3
hence we get fraction in simplest form as -22/3 .
the distance between earth and the moon is about 238,900 miles. round this number to the nearest ten thousand
Answer:
240000...............................
find the measure of angle B, note that angle B is acute
Answer:
B= 44.9506°
Step-by-step explanation:
WE apply sine angle formula
[tex]\frac{a}{sin(A)} =\frac{b}{sin(B)}=\frac{c}{sin(C)}[/tex]
Given angle C = 51, c= 11 and b = 10
[tex]\frac{b}{sin(B)}=\frac{c}{sin(C)}[/tex]
Plug in the values
[tex]\frac{10}{sin(B)}=\frac{11}{sin(51)}[/tex]
Cross multiply it
10 * sin(51) = 11* sin(B)
7.771459615 = 11 sin(B)
Now divide by 11 on both sides
0.706496328 = sin(B)
Now [tex]B = sin^{-1} (0.706496328)[/tex]
B= 44.9506°
Un circulo tiene un radio 3 yardas ,¿cual es el área ?
Answer:
28.27 yardas^2
Step-by-step explanation:
Area = pi r^2
= pi * 3^2
= 9 pi
or 28.27 yardas^2
Divide.
825÷(−215)
Enter your answer as a mixed number, in simplified form, in the box.
[tex]825\div(-215)=-\dfrac{825}{215}=-\dfrac{825:5}{215:5}=-\dfrac{165}{43}=-3\dfrac{36}{43}[/tex]
A pair of parallel lines is cut by a transversal. One of the angles formed measures 58°. Which statements about the other seven angles formed are true? (multiple answers)
There are three more angles with the same measure.
All the other angles have the same measure.
The rest of the angles measure 122°.
Only three of the angles measure 122°.
Four of the angles measure 122°.
Answer:
Correct options are 1 and 5.
Step-by-step explanation:
If parallel lines m and n are cut by a transeversal t, then corresponding angles are equal in measure. Therefore,
∠2≅∠6;
∠1≅∠5;
∠3≅∠7;
∠4≅∠8.
If parallel lines m and n are cut by a transeversal t, then interior angles on the same side are supplementary. Thus,
∠3+∠5=180°;
∠4+∠6=180°.
If one of the angles ( let it be angle 3) formed measures 58°, then m∠5=180°-58°=122°. Then
m∠2=m∠3=m∠6=m∠7=58°;
m∠1=m∠4=m∠5=m∠8=122°.
You can conclude that there are three more angles with the same measure of 58° and four of the angles measure 122°.
for a school fundraiser troy sold 28 bags of popcorn and 40 candy bars and made $282. Jake sold 17 bags of popcorn and 20 candy bars and made $160.50. What was the cost of the popcorn
To find the cost of the popcorn, set up a system of equations and solve them to find the value of x, which represents the cost of a bag of popcorn.
Explanation:To find the cost of the popcorn, we need to set up a system of equations. Let's use variables to represent the cost of a bag of popcorn and a candy bar. Let x be the cost of a bag of popcorn and y be the cost of a candy bar. We can set up two equations using the given information:
28x + 40y = 282
17x + 20y = 160.50
Now, we can solve this system of equations. Multiply the second equation by 2 to make the coefficients of x in both equations equal:
34x + 40y = 321
28x + 40y = 282
Subtract the second equation from the first equation:
34x - 28x + 40y - 40y = 321 - 282
6x = 39
Divide both sides of the equation by 6:
x = 6.50
Therefore, the cost of a bag of popcorn is $6.50.
Learn more about cost of popcorn here:https://brainly.com/question/188350
#SPJ2
Which is the net for this triangular prism?
Answer:
Step-by-step explanation:
a
f(1)=2 and f(n) = f(n-1) - 5; n-1
which of the following lists the terms in the sequence defined by this recursive function
a-2,3,8,13,18......
b-2,7,12,17,22.....
c-2,-3,-8,-13,-18.....
d-2,-7,-12,-17,-22....
Answer:
Option c is correct.
The sequence is, 2, -3 , -8 , -13, -18, .....
Step-by-step explanation:
Given the following as;
f(1) = 2
The recursive function is;[tex]f(n) = f(n-1) -5[/tex] ......[1]
to find the sequence defined by the recursive function;
Put n=2 in [1];
[tex]f(2) = f(2-1) -5[/tex]
[tex]f(2) = f(1) -5[/tex]
Substitute the value of f(1) = 2 we have;
[tex]f(2) = 2-5 = -3[/tex]
Similarly, put n =3
[tex]f(3) = f(3-1) -5[/tex]
[tex]f(3) = f(2) -5[/tex]
Substitute the value of f(2) = -3 we have;
[tex]f(3) = -3-5 = -8[/tex]
for n = 4 we have;
[tex]f(4) = f(4-1) -5[/tex]
[tex]f(4) = f(3) -5[/tex]
Substitute the value of f(3) = -8 we have;
[tex]f(4) = -8-5 = -13[/tex] and so on .......
Therefore, we get the sequence defined by the recursive formula is,
2, -3 , -8 , -13, -18, .........
What is
1.567 + 2.345 - 1.83(5) A) -2,082 B) -5.238 C) 2.082 D) 5.742
Answer:
c) 2.082
Step-by-step explanation:
First you would add 1.567 and 2.345 together to get 3.912. If you take 3.912 and subtract 1.83 from that, it would be 2.082.
Hey there!
1.567 + 2.345 - 1.83 (5)
1.567 + 2.345 = 3.912
3.912 - 1.83(5)
1.83(5) = 9.15
3.912 - 9.15 = -5.238
Answer: -5.238 ✅
Good luck on your assignment and enjoy your day!
~LoveYourselfFirst:)
Which of the following is closest to 19.9 times 199?
A. 4000 B. 3990 C. 3980 D. 3970 E. 3960
you only get 30 seconds for this question so no straight out multiplying! (NO CALCULATORS EITHER) i think the distributive property is used but I don't know how so please help meeeeeeeeeeeeeeeeeeeeeeeeeeeeeee
Answer:
E 3960
Step-by-step explanation:
Round 19.9 and 199 to 20 and 200
20*200 = 4000
Now subtract 20
20 * (200-1) = 4000-20
20 * 199 = 4000-20
20*199 = 3980
.1 of 200 is 20
so subtract 20
(20-.1) *200=3980-20
19.9 *200 =3960
(20-.1) *(200-1) = 3960
Find the length of the arc intercepted by a central angle of 240 degrees in a circle of radius r equals 9 feet
Answer:
37.7 feet ( to 1 dec. place )
Step-by-step explanation:
arc length = circumference × fraction of circle
= 2π r × [tex]\frac{240}{360}[/tex]
= [tex]\frac{2\pi (9)(240)}{360}[/tex] ≈ 37.7 feet